Stay ahead of learning milestones! Enroll in a class over the summer!

G
Topic
First Poster
Last Poster
Fill in a "magic" 6x6 square
v_Enhance   12
N Yesterday at 3:48 PM by zuat.e
Source: Taiwan 2014 TST3 Quiz 1, P1
Consider a $6 \times 6$ grid. Define a diagonal to be the six squares whose coordinates $(i,j)$ ($1 \le i,j \le 6)$ satisfy $i-j \equiv k \pmod 6$ for some $k=0,1,\dots,5$. Hence there are six diagonals.

Determine if it is possible to fill it with the numbers $1,2,\dots,36$ (each exactly once) such that each row, each column, and each of the six diagonals has the same sum.
12 replies
v_Enhance
Jul 18, 2014
zuat.e
Yesterday at 3:48 PM
Trapezium inscribed in a circle
shivangjindal   27
N Apr 2, 2025 by andrewthenerd
Source: Balkan Mathematics Olympiad 2014 - Problem-3
Let $ABCD$ be a trapezium inscribed in a circle $\Gamma$ with diameter $AB$. Let $E$ be the intersection point of the diagonals $AC$ and $BD$ . The circle with center $B$ and radius $BE$ meets $\Gamma$ at the points $K$ and $L$ (where $K$ is on the same side of $AB$ as $C$). The line perpendicular to $BD$ at $E$ intersects $CD$ at $M$. Prove that $KM$ is perpendicular to $DL$.

Greece - Silouanos Brazitikos
27 replies
shivangjindal
May 4, 2014
andrewthenerd
Apr 2, 2025
Intersection of a cevian with the incircle
djb86   24
N Mar 30, 2025 by Ilikeminecraft
Source: South African MO 2005 Q4
The inscribed circle of triangle $ABC$ touches the sides $BC$, $CA$ and $AB$ at $D$, $E$ and $F$ respectively. Let $Q$ denote the other point of intersection of $AD$ and the inscribed circle. Prove that $EQ$ extended passes through the midpoint of $AF$ if and only if $AC = BC$.
24 replies
djb86
May 27, 2012
Ilikeminecraft
Mar 30, 2025
Convex quadrilateral and midpoints [RMO2-2011, India]
Potla   14
N Mar 27, 2025 by mqoi_KOLA
Let $ABCD$ be a convex quadrilateral. Let $E,F,G,H$ be the midpoints of $AB,BC,CD,DA$ respectively. If $AC,BD,EG,FH$ concur at a point $O,$ prove that $ABCD$ is a parallelogram.
14 replies
Potla
Dec 31, 2011
mqoi_KOLA
Mar 27, 2025
IMO 2014 Problem 4
ipaper   167
N Mar 27, 2025 by bjump
Let $P$ and $Q$ be on segment $BC$ of an acute triangle $ABC$ such that $\angle PAB=\angle BCA$ and $\angle CAQ=\angle ABC$. Let $M$ and $N$ be the points on $AP$ and $AQ$, respectively, such that $P$ is the midpoint of $AM$ and $Q$ is the midpoint of $AN$. Prove that the intersection of $BM$ and $CN$ is on the circumference of triangle $ABC$.

Proposed by Giorgi Arabidze, Georgia.
167 replies
ipaper
Jul 9, 2014
bjump
Mar 27, 2025
geometry coordinates
CHESSR1DER   0
Mar 20, 2025
Source: simplified version of Belarus TST
Points $A, B, C$ with rational coordinates lie on a plane. It turned out that the distance between every pair of points is an integer. Prove that there exist points $D, E, F$ with integer coordinates such that $AB = DE$, $AC = DF$, $BC = EF$
0 replies
CHESSR1DER
Mar 20, 2025
0 replies
Folding...
Rushil   14
N Mar 7, 2025 by Jupiterballs
Source: RMO 1990 Problem 3
A square sheet of paper $ABCD$ is so folded that $B$ falls on the mid point of $M$ of $CD$. Prove that the crease will divide $BC$ in the ration $5 : 3$.
14 replies
Rushil
Oct 14, 2005
Jupiterballs
Mar 7, 2025
Nearby lattice point with relatively prime coordinates
MellowMelon   3
N Mar 7, 2025 by quantam13
Source: USA TSTST 2011/2012 P3
Prove that there exists a real constant $c$ such that for any pair $(x,y)$ of real numbers, there exist relatively prime integers $m$ and $n$ satisfying the relation
\[
\sqrt{(x-m)^2 + (y-n)^2} < c\log (x^2 + y^2 + 2).
\]
3 replies
MellowMelon
Jul 26, 2011
quantam13
Mar 7, 2025
Rotating segment by 45 degrees and interchanging endpoints.
Goutham   9
N Mar 6, 2025 by Mathandski
A needle (a segment) lies on a plane. One can rotate it $45^{\circ}$ round any of its endpoints. Is it possible that after several rotations the needle returns to initial position with the endpoints interchanged?
9 replies
Goutham
Feb 9, 2011
Mathandski
Mar 6, 2025
A hard coordinate problem
Dftioscjg   4
N Feb 28, 2025 by jasperE3
A chip moves in a plane provided with an orthonormal coordinate system.
Initially the chip is located at the point of coordinate (0,0). The chip can only perform jumps : of length 5 units, in a straight line and in any direction. Moreover, the chip can only reach points whose two coordinates are integers. The chip wants to reach the point of coordinates(2023,0).
What is the minimum number of jumps she will do?
4 replies
Dftioscjg
Feb 11, 2025
jasperE3
Feb 28, 2025
a